Board logo

标题: 请教LSAT20_II_7,8 [打印本页]

作者: joywzy    时间: 2003-1-20 17:25     标题: 请教LSAT20_II_7,8

7. Brownlea's post office must be replaced with a larger one. The present one cannot be expanded. land near the present location in the center of town is more expensive than land on the outskirts of town. Since the cost of acquiring a site is a significant part of the total construction cost, the post office clearly could be built more cheaply on the outskirts of town.

Which one of the following, if true, most seriously undermines the argument's stated conclusion?

(A) The new post office will have to be built in accordance with a demanding new citywide building code.

(B) If the new post office is built on the outskirts of town, it will require a parking lot, but if sited near the present post office it will not.

(C) If the new post office is built on the outskirts of town, current city bus routes will have to be expanded to provide access.

(D) If the new post office is built on the outskirts of town, residents will make decreased use of post office boxes, with the result that mail carriers will have to deliver more mail to homes.

(E) If the new post office is built near the center of town, disruptions to city traffic would have to be minimized by taking such steps as doing some construction work in stages at night and on weekends.

Ans: B
i often made mistakes in weaken . For example, this question, i think A(他因削弱)\B\C(扩展公交=>多花钱,削弱)都可以削弱.我究竟错在哪呢? 我经常发现几个选项都可以削弱,然后不知选哪个好.

8. in the past, the railroads in Ostronia were run as regional monopolies and operated with little regard for what customers wanted. In recent years, with improvements to the Ostronian national highway network the railroad companies have faced heavy competition from long-distance trucking companies. But because of government subsidies that have permitted Ostronian railroad companies to operate even while incurring substantial losses, the companies continue to disregard customers' needs and desires.

If the statements above are true, which one of the following must also be true on the basis of them?

(A) If the government of Ostronia ceases to subsidize railroad companies. few of those companies will continue to operate.

(B) Few companies in Ostronia that have received subsidies from the government have taken the needs and desires of their customers into account.

(C) Without government subsidies, railroad companies in Ostronia would have to increase the prices they charge their customers.
(D) The transportation system in Ostronia is no more efficient today than it was in the past.

(E) In recent years, some companies in Ostronia that have had little regard for the desires of their customers have nonetheless survived.

Ans: E ,but i choose B
我觉得E中提到survive,C中提到increase the price, D中提到efficient,都是原文没有出现的概念,所以答案只能在A,B中选,我觉得B更好点.不知错在哪?




欢迎光临 国际顶尖MBA申请交流平台--TOPWAY MBA (http://forum.topway.org/) Powered by Discuz! 7.2